Auto 122 Exam 41

Ace your homework & exams now with Quizwiz!

How much current will flow in this circuit?

2 amps

If a 12-volt battery is connected to a series circuit with three resistors of 2 ohms, 4 ohms, and 6 ohms, how much current will flow through the circuit?

1 Amp

Calculate the total resistance and current in a 12 volt parallel circuit with the three resistance of 4 ohms, 8 ohms, and 16 ohms, using any one of the five methods(calculate suggested). What is the total resistance and current?

2.3 Ohms (5.3 amperes)

What is the total current flow in this parallel circuit?

3.7 A

What is the total resistance in this circuit?

6 ohms

Calculate the current flow in this circuit. (Round off to 2 decimal points.)

About 0.95 amps

Which one of the bulbs in this circuit will drop (use up) the most voltage?

Bulb 1

In a series circuit..............

Current flow is constant at any point in the circuit Total circuit resistance is equal to the sum of all resistance in the circuit Both A and B

As additional branches are added to a parallel circuit resistance.....................

Decrease

Knowledge of parallel circuit fundamentals is necessary to diagnose....................

Diesel glow plug circuits Add-on lighting circuits Port fuel injection circuits All of these

The sum of currents in each branch of a parallel circuit will.................total circuit current.

Equal

As additional branches are added to a parallel circuit, total circuit amperage...................

Increase

The total circuit resistance of a parallel circuit is always..................the lowest resistance present in any branch of the circuit.

Less than

More electrical current will tend to flow through the branch of a parallel circuit with................resistance.

Lowest

If resistance is reduced in a series circuit,.....................current will flow.

More

What type of circuit is this?

Series-parallel circuit

Technician A says that electronic calculators have a memory function that can be used to calculate resistance, current , and voltage value for a parallel circuit. Technician B says to separate components of a Series-parallel circuit to simplify calculations. Which technician is correct?

Technician A only Technician B only Both technician A and B

In a parallel circuit.....................

current flow through each branch of the circuit depends on the resistance present in that branch The is more than one path for current flow Both A and B

The amperage in a series circuit is ..........................

The same anywhere in the circuit

If one branch of a parallel circuit loses continuity, the other branches will still each receive source voltage and ground.

True

Kirchhoff's law states that the voltage around a closed circuit is equal to the sum of........................across.

Voltage drops

What is the total resistance of this circuit?

4 ohms

The voltage drop for each branch of a parallel circuit is..............................

Equal

The current flowing into each junction of a parallel circuit................the current flow at the junction on the opposite end of that branch.

Equals

If the resistance and the voltage are known, what is the formula for finding the current(ampere)?

I=E/R

An electrical load...................

May be an electrical device. Has electrical resistance May be unwanted resistance caused by corrosion All of these

In a series circuit with more than one load, the highest resistance will drop....................... voltage than any smaller resistance.

More

The sum of the voltage drops in a series circuit equals the.........................

Source voltage

In a series circuit, total circuit resistance is equal to the..................of the resistance of all loads in the circuit.

Sum

A series circuit has three resistors of 4 ohms each. The voltage drop across each resistor is 4 volts. Technician A says that the source voltage is 12 Volts. Technician B says that the total resistance is 18 ohms. Which technician is correct?

Technician A

A series circuit has two 10-ohms bulbs. A third 10-ohm bulb is added in series. Technician A says that the three bulbs will be dimmer than when only two bulbs were in the circuit. Technician B says that the current in the circuit will increase. Which technician is correct?

Technician A only

Technician A says that the sum of the voltage drops in a series circuit should equal the source voltage. Technician B says the current (ampere) varies depending on the value of the resistance in a series circuit. Which technician is correct?

Technician A only Technician B only Both technician A and B

Technician A says that Ohms law can be used to determine circuit current flow if total circuit resistance and total voltage are known. Technician B says that Ohms law can be used to calculate the unknown resistance of a load in a circuit if total current and source voltage are known. Which technician is correct?

Technician A only Technician B only Both technicians A and B

Two light bulbs are wired in series and one bulb burns out (opens.) Technician A says that the other bulb will still work. Technician B says that the current will increase in the circuit because one electrical load (resistance) is no longer operating. Which technician is corrent?

Technician A only Technician B only Neither technician A and B

A vehicle has four parking lights all connected in parallel and one of the bulbs burns out. Technician A says that this could cause the parking light fuse to blow (open). Technician B says that it would decrease the current in the circuit

Technician B Only

Two bulbs are connected in parallel to a 12-volt battery. One bulb has a resistance of 6 ohms and the other bulb has a resistance of 2 ohms. Technician A says that only the 2-ohm bulb will light because all of the current will flow through the path with least resistance and no current will flow through the 6-ohm bulb. Technician B says that the 6-ohm bulb will be dimmer than the 2-ohm bulb. Which technician is correct?

Technician B only

Voltage drop is a better measurement of resistance than testing static resistance because.......................

The resistance specification for the component is not required for diagnosis Current flow creates heat which adds to circuit resistance Both A and B


Related study sets

LENB 3135 Chapter 19 Study Questions

View Set

Perry- Chapter 48: Musculoskeletal or Articular Dysfunction

View Set

unit 8 The media phrasal verbs exercises

View Set

Operations Management Exam 2 Homework Questions

View Set

Episodic & Semantic Memory (Chapter 7)

View Set